Sequence Convergence & Fixed Point Theorem

Click For Summary
The sequence defined by pn = g(pn-1) converges for any initial value p0 in the interval [1, 2], as g(x) is continuous and satisfies the conditions of the Fixed Point Theorem. The derivative g'(x) is continuous on [1, 2] and remains less than 1, confirming convergence. For determining the rate of convergence, the Taylor series expansion around the fixed point p can be used, focusing on the first two terms to analyze error reduction at each iteration. The error is expected to decrease by a constant factor a with each step, indicating the convergence behavior. Understanding these principles is essential for analyzing the sequence's convergence properties effectively.
f00lishroy
Messages
6
Reaction score
0

Homework Statement



Let g(x)= (2/3)*(x+1/(x^2)) and consider the sequence defined by pn= g(pn-1) where n≥1

a) Determine the values of p0 \in [1,2] for which the sequence {pn} from 0 to infinity converges.

b) For the cases where {pn} converges (if any), what is the rate of convergence?


Homework Equations



http://en.wikipedia.org/wiki/Fixed-point_theorem

Fixed Point Theorem

attachment.php?attachmentid=58589&stc=1&d=1368037754.png



The Attempt at a Solution



For part a, my answer is that ANY point p0 between 1 and 2 will converge, because the sequence satisfies the fixed point theorem.


g(x) exists on [1,2] and is continuous
g'(x) = (2/3)(1-2/(x^3)) exists and is continuous on [1,2]

There is a positive constant k<1 for which |g'(x)|≤k

By plotting g'(x), i found k = 2/3

Therefore, the fixed point theorem is satisfied, and so should the answer be "any value of p0 from 1 to 2 will cause the sequence to converge"?

For part b, I am not sure how to find the rate of convergence. I heard that you have to take the taylor series expansion, but I am not sure. Any help? Thanks
 

Attachments

  • Screen Shot 2013-05-08 at 11.29.43 AM.png
    Screen Shot 2013-05-08 at 11.29.43 AM.png
    11.1 KB · Views: 665
Physics news on Phys.org
Yes, that shows it converges throughout the interval.
For the rate, let p be the fixed point and x be small. If the rate of convergence is a then g(p+x) is approximately p + ax, i.e. the error gets multiplied by factor a at each step. Take the Taylor expansion of g at p, just the first two terms, and see what that simplifies to.
 
Question: A clock's minute hand has length 4 and its hour hand has length 3. What is the distance between the tips at the moment when it is increasing most rapidly?(Putnam Exam Question) Answer: Making assumption that both the hands moves at constant angular velocities, the answer is ## \sqrt{7} .## But don't you think this assumption is somewhat doubtful and wrong?

Similar threads

  • · Replies 5 ·
Replies
5
Views
2K
  • · Replies 1 ·
Replies
1
Views
2K
Replies
1
Views
1K
  • · Replies 2 ·
Replies
2
Views
2K
  • · Replies 4 ·
Replies
4
Views
2K
  • · Replies 7 ·
Replies
7
Views
2K
  • · Replies 16 ·
Replies
16
Views
1K
  • · Replies 6 ·
Replies
6
Views
3K
  • · Replies 2 ·
Replies
2
Views
2K
  • · Replies 1 ·
Replies
1
Views
1K